« first day (1795 days earlier)      last day (3229 days later) » 
00:00 - 17:0017:00 - 00:00

12:06 AM
@TedShifrin So for the fourth problem, should my proof involve eigenvalues? Would something like that work? If not, why?
 
Does anybody know when @Semiclassical comes?
 
@pjs36: This is the second copy of that post. It's complete nonsense, and there's no reason to believe he understands what any of the words mean.
 
12:24 AM
wow what the hell did I just read
 
I was going to say 'warning for strong language and racism' but really, just, warning
 
Oh, @MikeMiller, I'd discerned that he was indeed crazy (I voted to close the first, by the way). But, you're right - it was a silly thing to do.
But, thanks for the link - I truly underestimated the extent of insanity!
 
Most of it sounds made up.
 
(this is the relevant MSE post - if you have the rep, vote to kill with fire)
3
 
Just looking for attention/sympathy
 
12:32 AM
Ah, I forgot that voting "for great justice" actually accomplishes that!
 
I don't get anything he's saying
 
Alert the mods, there's another psycho-troll on the loose.
 
12:56 AM
@MikeMiller dunno offhand
 
Hi @pjs36 ... Your silly edit noted :)
@Stan: Which was the fourth problem?
I am not psycho, @skull.
 
1:14 AM
Hi, I have a simple question that is probably not worth posting to the main site. So if I and J are ideals, with $I\subseteq J$, then clearly $rad(I)\subset\rad(J)$, where rad means radical. What about the converse? If $\rad(I)\subset\rad(J)$, would this imply that $I\subset J$? I am guessing no, but can't seem to find a counter-example.
 
1:28 AM
Nah, @Prism, I don't think so. What have you tried?
 
@anon: any idea where I should look?
 
nope
google says there's a book about it
actually not sure if the book talks about existence
 
what's the book
 
SL(2) representations of finitely presented groups
 
also, could you unstar the 'he's insane' link, it's got content i don't really think should be on the starboard
will look, thanks
danke
 
1:43 AM
it doesn't seem relevant (unfortunate given the title)
 
focused on the modular case or something?
 
@TedShifrin: Well, if $f\in I$, then $f\in rad(I)$, so $f\in \rad(J)$, so $f^{n}\in J$. So if we can manage that $n\geq 2$ (and $n$ no lower), then we get our counter-example.
 
2:03 AM
Is $x\in \text{rad}((x^2))$? @prism
 
@TedShifrin: I am so dumb… Thanks a lot :)
 
Sure :)
 
@TedShifrin hey ted!
 
Hi @Stan
 
The fourth problem involved the Hessian
And I believe I have to show it is negative definite
 
2:17 AM
Oh, the chain rule problem?
 
@TedShifrin the second derivative test one
 
Right. And I told you to parametrize and use the chain rule.
 
Hi guys!
What's the problem?
 
@TedShifrin okay, let me think about that then and ping you tomorrow about it. I wanna try that out.
 
Ok.
 
2:30 AM
Can a Peano curve be a bijection?
e.g. the one $[0,1]\to[0,1]^2$.
I think so: simply "flatten" the curve and contract it to have unit length.
Place it alongside $[0,1]$ and identify. Does this work?
 
what?
 
Is the Peano curve from the closed unit interval to the unit square a bijection?
I think it shouldn't be, but why is what I'm saying wrong?
 
I don't know what you're saying.
 
Uh.
Can space-filling curves be continuous bijections?
Is that clear enough?
 
I know what your question is. What I couldn't make sense of was your "flatten and contract the curve" comment.
BTW, try googling "Can space-filling curves be continuous bijections?"
 
2:55 AM
how do I do mathfrak by hand?
 
my solution is not to try usually
 
welp
why is this even used in the literature
 
spite
 
"I know, I'll let $\mathfrak{p}$ be a prime ideal and let $p \in \mathfrak{p}$. Let's see sam yusim wrangle his way out of this one!"
 
well, it's not spite for you specifically
the more precise question is 'why did the Germans start writing in fraktur'
 
3:00 AM
well like replace my name with you or whoever else
 
and the answer to that is spite for humanity in general
 
it's probably not wrong to blame hilbert for this
 
r9m
@Chris'ssistheartist seems he has changed his profile name to his real name .. (sos440 I mean) :-)
 
3:16 AM
@anon Well, since the curve is just a very curled-up line, we can "grab" the two extreme "ends" of the curve and straighten it out like a thread, is what I was saying. But it won't work, as your link says.
 
Hi @robjohn
 
4:02 AM
@skillpatrol Hey there... anything planned for tomorrow?
 
@Rem, did you do the Hausdorff $\iimplies$ set of fixed points closed problem yet?
 
Huy
4:24 AM
Good morning, @MikeMiller.
 
Hi @Huy.
 
Nothing special @robjohn how about you?
 
Hi @Robjohn
Need some help from Integration experts : math.stackexchange.com/q/1348843/2987
2
 
@MichaelAlbanese: OK, I've finally got a result I'm satisfied with. We have the nice form $\text{Diff}(S^n) \simeq O(n+1) \times \text{Diff}(D^n, \partial D^n)$. So we see all the complication comes from this second factor. It'd be nice to have a similar thing for Homeo. I think I've finally found it but I don't have access to all the papers I need right now, I'm writing them down so I can take a look when I get home.
 
@Mike, do you get access to papers at home as well?
 
4:40 AM
Here's a taste. Cerf proved that $\pi_{i+m+1}(\text{Homeo}(S^m),O(m+1)) \simeq \pi_i(\text{Diff}(S^m),O(m+1))$. He also calculated the smaller homotopy groups of the former. I think (hope?) this really takes the form of something like $\text{Homeo}(S^m) \simeq O(m+1) \times \Omega^m \text{Diff}(D^n,\partial D^n)$. But in particular this tells us the homotopy of $(\text{Homeo}(S^m),O(m+1))$ is also wild for $m \geq 5$, and in the same way!
When I get a chance I'm going to try to take a look at the papers this comes from to get a more precise version, and a taste for why it's true. I don't know if I'll have time; hopefully it's not too wild.
@SohamChowdhury: I'm traveling right now. I meant when I get back to the university. I should be able to set up a VPN to get journal access even when I'm away but I forgot to.
 
@RajeshD Just saw your question. Suggested some clarification
 
I don't understand some statement in my mechanics book
not sure if my understanding is correct
 
5:08 AM
got it corrected @robjohn
 
@MichaelAlbanese: I have a sign wrong up there. I really want to write $O(m+1) \times \Omega^{-m} \text{Diff}(D^n, \partial D^n)$, where by that I mean some space whose $m$th loop space is the desired object. It'd also be nice if there were a concrete descriptor of this delooped space.
 
6:03 AM
wow, chat is dead.
 
6:24 AM
1
Q: Change of order of integration of a triple integral

Rajesh DConsider $$ I = \int_0^{\omega}\int_0^{\alpha}\int_0^{\alpha}F(\beta){\tilde{F}(\gamma)}e^{i\beta t}e^{-i\gamma t}R(\alpha)d\beta d\gamma d\alpha$$ In this triple integral,I want to bring about, a change of order of integration, where in I take integration with respect to $\alpha$ inside most an...

chat is alive
 
r9m
7:14 AM
@robjohn could you take a look at part $(2)$ in this question when you have time? :-)
 
8:00 AM
@BenDover I "know" what it is, but I have no idea why should I care about it
I have no idea why should I care about category theory either
@SohamChowdhury Peano curves are not injective.
In particular, there are no continuous bijection from $[0, 1]$ to $[0, 1]^2$, iirc.
 
@r9m I see. :-)
 
9:04 AM
Hello
 
@Hippalectryon I wanna show you something.
 
@Chris'ssistheartist sure !
 
BBL (60-90 min)
 
9:36 AM
@Semiclassical I dont understand exactly... you left, some time ago, a link for exactly I was searching for, yes. I leave a comment here but I dont know if you read it. Yes, it is very interesting, thank you very much.
 
10:01 AM
@BalarkaSen Yeah. If there were, then I guess those two would be homeo, which they are not (because the unit interval minus pt is not connected, although I can't prove that yet).
@BalarkaSen . . .
 
@r9m or at least a name.
 
@SohamChowdhury No, why would that mean it's a homeo?
@SohamChowdhury ?
 
Continuous + bijective. I think so, not at all sure.
 
homeo means continuous bijective and continuous inverse
I don't recall how to prove it either, but I am not willing to think about it right now.
 
yeah, okay, thanks.
 
10:16 AM
did you have fun with p-adics yesterday?
 
oh, yes. I'll write up what I learned to cement my understanding.
 
definitely
 
@SohamChowdhury The Hilbert and Peano curves from $[0,1]\to[0,1]^2$ are Hölder-$\frac12$ continuous, and not rectifiable. This means you cannot straighten them out to unit length.
 
Ah, that is nice to know; I was wondering why I couldn't. :)
 
10:26 AM
@SohamChowdhury This is mentioned in the sixth bullet here
 
10:55 AM
@SohamChowdhury Do you mean the diagonal being closed?
 
Was that your question?
 
Which one do you mean ? @Soh
 
@SohamChowdhury yeah, he did figure it out.
 
but it could be done in a more straightforward way
 
10:59 AM
Diagonal in a Hausdorff space is closed. It's quite simple, really.
 
no, you don't need that.
 
?
That is the thing we have to prove.
oh, sorry, I was thinking about something else.
 
here is the proof without using that diagonal argument.
sorry : the question was to prove that a function from a set extends to a function from the closure uniquely.
 
wait. I have no clue how that relates to the problem I was talking about.
anyway.
 
yep, i misread that.
he also figured out the hausdorff => finite set closed problem.
 
11:04 AM
I really like the finite subcover def of compactness.
(I've said that before)
 
it's a cute definition, sure.
 
Its really damn cool.. the hausdorff one
Hello@Balarka
 
@Remember Soham is talking about the definition of compactness, not hausdorffness.
oh, turns out my analogy for fibers is right - prof just emailed me. phew.
 
But I am talking about the definition of Hausdorffness and diagonal @Balarka
 
I don't like the diagonal definition.
you lose the geometric intuition behind Hausdorfness.
 
11:07 AM
Yes, thats true
But since I am on the last part of first chapter ... I am liking quotient spaces a lot though
 
read quotient spaces carefully.
it's a subtle thing.
 
Yes.... it is just a more tighter definition of continuity right ?
 
huh
 
@BalarkaSen geometric intuition is about disjoint nbhds, right?
 
what does quotient spaces have to do with continuity.
@Soham right
 
11:10 AM
there are other defs?
 
sure.
 
oh, ok.
 
Not quotient spaces ... I mean the definition of quotient map@Balarka
 
there are two proofs of Heine-Borel in this book. both are pretty interesting and feel like important strategies.
 
I feel my next chapter is more interesting @Soham Is about local compactness and connectedness
 
11:12 AM
local connectedness?
 
Interchanged it :p
 
what is local compactness?
googled, nvm
 
@Remember i don't what the quotient map says about continuity.
 
I think path connectedness will also be there
They dont say anything about continuity but its definition is lot similar to the continuity definition ...
 
not at all
 
11:14 AM
Its also called (for that reason ) strong continuity
 
really?
 
Given in Munkres
 
I didn't know that. I don't know what it has to do with continuity.
It's just an analogue of the quotient map in the algebraic case.
 
Okay brb
 
identification spaces indeed felt very similar to quotient groups and categories when I looked at them last time, @Balarka. can't wait to hit them.
 
11:18 AM
just don't rush. yes, they are similar to quotient groups.
 
I expect there is some categorical explanation (which you don't care about?) for that that I'll learn about later. :P
 
11:38 AM
@Balarka, I think he was talking about path-connectedness. ("strong continuity")
 
@SohamChowdhury I don't think so.
Path-connectedness is a intristic property of the topological space you are working with (i.e., two points can always be joined with a path). What has that to do with continuity (which is a property of a map)?
 
ah, yes, you're right.
 
hmm, now that you say it, I see why it's called strong continuity.
@SohamChowdhury right.
 
sort of a converse to the "preimage of open sets is open", I think?
 
that definition is actually not my definition of a quotient map, thus the confusion :)
 
r9m
11:46 AM
@robjohn he's there on my facebook friend list .. I doubt the name he put on fb (the same as the one that is currently his M.se alias) is not his real one too .. :|
 
@SohamChowdhury What're you studying right now?
 
the second proof of Heine-Borel.
 
ok. topology on $\Bbb R^n$ is a bit boring (no offense :P)
 
don't rub it in.
i want to get over this messy stuff asap.
I like cooking, not cutting the heads off chickens. :P
 
haha, make sure you don't.
I think that was ambiguous. I meant "don't get over the messy stuff asap".
 
11:55 AM
Einstein once said something like "make your theories as simple as possible, but not simpler."
So, similarly, asap, but not sooner.
what proof of Heine-Borel do you know?
 
We've already had that discussion before. The proof I know by heart is using Tychonoff.
 
have we? oh.
 
The Bolzano-Weierstrass proof I have forgotten, but I can reconstruct it given some time.
@Soham Fancy application of Tychonoff : $\mathbf{Z}_p$ is compact.
 
because subspace of infinite product space?
(idk what I'm talking about, just remember from some preface that Tychonoff is about compactness of infinite products)
 
it's a subspace of $\prod \Bbb Z/p^k\Bbb Z$, which is compact by Tychonoff. but you need more work than this, as it's not nessesary that subspaces of compact things are compact.
 
11:58 AM
hmm.
 
tychonoff says product of compact spaces are compact.
 
@Hippalectryon Mamma mia what an amazing proof I have to this one mathworld.wolfram.com/HadjicostassFormula.html. :-) You'll find that in my book. And also for $(4)$.
 
@BalarkaSen cool
 
@Chris'ssistheartist :D
 
12:00 PM
the correct thing is that closed subspace of compact spaces is compact.
so you need to show that Z_p is compact in the product. some work is needed to do this.
 
@Hippalectryon It's something like OOOO MYYYY GOOODDD! :-)))))))))
(laughing here :-)))))
 
I guess you can use that it's complete. No idea.
 
interesting. future attractions :)
 
@hippa
 
ah, finished the second proof as well. both were really cool.
 
12:05 PM
...as soon and as simple as possible; not sooner and not simpler ;-)
 
@BalarkaSen so how come you even use the notion of a cateogry
11
 
I don't recall if I have ever used categories anywhere in my work :P
 
@r9m I have posted an answer to the first sum... with my recursion, I might be able to get the second.
 
@Hippalectryon The approach I have there is like an A-bomb to the problem, simple, but at the same time profound and deadly. :-)
 
@Chris'ssistheartist Nice :-)
 
12:07 PM
:D
 
:D
@skillpatrol Thanks. I know how to write it but I missed a letter. :D
 
You're going to be a published author soon, so all typos are your responsilbity pal :-)
 
@Balarka, the book proves BW later using HB. :P
so I guess you know a third proof.
 
you don't need HB to prove BW
I know of a proof that doesn't need HB.
 
*responsibility
 
12:15 PM
@skillpatrol ouch :P
 
@Soham actually, you can take that as an exercise. prove BW by hand.
it's an analysis-fact. no need to know about compactness at all.
ok, I have to go.
 
later pal
 
@Hippalectryon show me a brilliant solution to $$\int_0^{\log \left(1+\sqrt{2}\right)} \operatorname{arccot}(\cosh (x)) \, dx$$
 
@r9m I got the second one, too
 
@Soham Heine Borel makes you feel you are superman !! :p
 
12:29 PM
@SohamChowdhury :D
 
@Chris'ssistheartist: I don't know if you would be interested in my latest answer, but it does use some interesting sums. I need to shorten my derivation of the recursion I use.
 
@robjohn Nice (+1)
 
@Chris'ssistheartist I've checked it out in Mathematica for $n\le5$ and they both work, so I am pretty sure I didn't make any errors. :-)
 
@robjohn Interesting that $(9)$ relation.
@robjohn the idea in $(2)$ is something that one should always check. Good stuff.
 
@Chris'ssistheartist At some point I should verify that r9m's answer for the first one matches mine.
@Chris'ssistheartist I am working on making my presentation of $(2)$ shorter
 
12:40 PM
@robjohn I'm curious how you do that. :-)
@r9m I saw you above talking about identities referring to sos, and the funny thing is that many of the people I talk to they think I'm 50-60 years old (referring to Romanian). That's pretty funny, and many refuse to believe my real age. :-)))
I was talking last night with a cute person who told me that I might probably be 60-65 years old, but maybe also because of my poetic speech in Romanian. :-)
 
"who told me" @Chris'ssistheartist
 
@Rememberme OK :-)
Actually after a certain point almost all our talk was exclusively about age, and less about other things that might have been far more interesting. :-)
Anyway.
@robjohn I can finish both identities nicely, in my style.
 
r9m
1:07 PM
@robjohn (+1) That was super cool!!! :D
 
@Chris'ssistheartist I would like to see your derivations of them.
 
@robjohn Stuff similar to what r9m posted. I didn't put anything on paper yet, but I see the way to go without any problem, difficulty.
 
@r9m I was glad that Mathematica said they agreed for $n\le5$; it meant that I probably hadn't made any mistakes :-)
 
r9m
@robjohn okay! did I make some mistake? I'm unable to see how $\sum\limits_{m=1}^{n}\frac{H_{m+n}}{m} - H_n^2 = \frac{1}{2}H_n^{(2)}$
 
@r9m I don't know... I haven't checked your answer in Mathematica yet.
 
1:14 PM
@r9m Maybe you wanna use the summation by parts.
$$\sum\limits_{m=1}^{n}\frac{H_{m+n}}{m}=\sum\limits_{m=1}^{n}\frac{H_{m}+\frac{‌​1}{m+1}+\frac{1}{m+2}+\cdots+\frac{1}{m+n}}{m}$$ and then split. You have the well-known sum $\sum\limits_{m=1}^{n}\frac{H_{m}}{m}$ and the rest part to be done by sumamtion by parts.
 
@Rememberme not really; @Chris'ssistheartist's use of "that" is perfectly standard in AmE.
 
@SohamChowdhury I wasn't sure about that. Thanks. :-)
 
:)
Oh my god, this book uses $\phi$ for the empty set. ;-;
Why does every introductory topology book have a hamartia? >:-(
 
Whats the problem with it? @Soham
The empty set notation
 
That's a $\phi$, not a $\emptyset$.
 
1:34 PM
hello ted
 
Hi, bananas
 
i'm doing some complex analysis
in particular, im trying to determine the residue at each isolated singularity of $f(z)=\frac{(z-1)^2}{(e^z-1)^3}$
 
I just compute the relevant part of the Laurent series
 
this function has poles of order 3 at $2k\pi i$ for each $k\in \Bbb{Z}$, right?
 
Yes.
 
1:38 PM
@TedShifrin well, how do you compute the relevant part of the laurent series?
 
Put $u=x-2\pi ki$ and use basics ...
What's the denominator?
 
@Ted, help.
 
$z+z^2/2 + z^3/6+...$
 
How do we know that there's a finite subcover which includes $X-C$?
 
Not at $0$, bananas ...
 
1:42 PM
That is, compactness of X ensures that for any open cover of X, there is a finite subcover. Why must such a subcover contain $X-C$?
 
I don't understand, Soham. What is your question?
Sure, because you have to cover all of $X$.
 
I might end up doing that with different open sets . . . oh, but their union is gonna be $X-C$. Right.
Dunno, @Ted, this feels sort of tautological.
 
If your $O_j$ are so huge that they cover $X-C$, so what?
 
I get it now. I think I was thinking "disjoint" all along for some reason.
 
Almost never ... Unless your space is very disconnected.
 
1:47 PM
So a finite subcover of some $A$ can "exceed" $A$, right?
 
Yes.
 
This makes the theorem so tautological . . .
Let $X$ be a compact space and $A$ a closed subset. Take any open cover of $X$. By compactness, there is a finite subcover. This finite subcover also covers $A$. Done.
 
Huy
@TedShifrin: I actually think cutting the solid horizontally is the easiest method. It's not that complicated once I actually did it.
 
Hi, @Huy.
 
No. You start with a cover of $A$. By definition of subspace topology, this gives you open sets in $X$.
 
Huy
1:49 PM
hey @Soham
 
Ok, @Huy.
 
@TedShifrin And then I intersect them with $A$ to get a finite cover of $A$ (in the subspace topology)?
 
Huy
@MikeMiller: What was that MSE post that you linked in the star board?
 
@Soham, make sure you understand that it's essential for $A$ to be closed.
 
@TedShifrin I do now. Otherwise $X-A$ won't be open, so the argument falls through.
 
1:52 PM
Right.
 
Yes @Soham anon gave me a nice hint and I solved it with it.. I mean your question
 
Thanks.
@Rememberme which one?
 
@Ted Howdy?
 
Howdy.
 
This open cover one.. I remember doing it when I did Rudin @Soham
 
1:54 PM
How far did you get?
 
I started with open cover for A but wasn't able to go much far :)... I got stuck after defining an open cover and then anon gave me a hint of the subspace thingy.. and I chose the values according to it (X-A here) which gave me an open cover of X@Soham
@Soham I am really irritated today... All these dumb educational institutions are playing with my photos
 
Phitji?
 
Aakash, EduTalk and all
I switched classes okay from one institution to another since I was getting free educations and some paid .... Now when I got some marks in my examination they are using my photos in their pamphlets ...... Unknown institutes are also coming from nowhere@Soham
And every other guy comes to my house saying ... Hey you are the topper right and all sorts of nonsense ... tell me how should I study?, what books did you refer....
Ahhhh..............
I hate this!!
@Soham Have you done quotient topology?
 
@robjohn @r9m use this math.stackexchange.com/questions/919572/… to finalize the question.
$$\sum\limits_{m=1}^{n}\frac{H_{m+n}}{m}=\sum\limits_{m=n+1}^{2n}\frac{H_{m}}{m-‌​n}$$
 
2:13 PM
@Rememberme Not yet. I got a full scholarship to FIITJEE and refused it (my condition for taking the "talent exam" was that I wouldn't join, lol)
So you're a topper? Nice.
 
No I am not please don't call me that
@Soham Quotient topology is pretty intuitive....
@Soham You have done algebra till Galois theory?
 
r9m
@Chris'ssistheartist I must have made some mistake ... my answer does not match for $n=2$ for example
 
@Rememberme Haha, not at all.
I barely know a little group theory, is all.
 
Rings?
 
Not at all. You?
 
2:15 PM
:22561512I checked that for larger values and it works.
 
Nope......
I would love to
 
Okay, got to go.
 
@Soham Peace!!
 
@r9m it works also for $n=2$
 
r9m
@Chris'ssistheartist oh! now I wonder what I'm doing .. can't figure out where I made the mistake .. :(
 
2:19 PM
@r9m $$\sum _{m=1}^2 \frac{H_{m+2}}{m}-\left(H_2\right){}^2-\frac{H_2^{(2)}}{2}=0$$
 
r9m
@Chris'ssistheartist oh! right ... I forgot the $\frac{H_n^{(2)}}{2}$ part and messed up .. :( sorry
 
@Chris'ssistheartist @r9m I finally shortened the proof of $(2)$. It is not too bad now.
The original derivation did not have the goal in mind, so it was much longer
 
r9m
@robjohn nice !!! :-)
 
@robjohn nice
 
@r9m You deleted your answer? Are you working on the $4^\text{th}$ power one?
 
r9m
2:35 PM
@robjohn I can't figure out how to show that $\sum\limits_{m=1}^{n}\frac{H_{m+n}}{m} - H_n^2 = \frac{1}{2}H_n^{(2)}$ .. (that is if at all what I did is correct)
 
r9m
2:54 PM
@Chris'ssistheartist ah!!!! I'm done with the step you showed here ... Thanks a lot!!!! :D
 
r9m
3:06 PM
@Chris'ssistheartist @robjohn edited and undeleted my answer .. thanks!! :-)
 
@Huy Crank post about the Hodge conjecture. Total nonsense. Second post by the same guy on the subject. I think he has a question ban now, since he hasn't posted again.
 
@r9m That is the easiest way I think. It would be a nice contest question for high school. :-)
 
r9m
@Chris'ssistheartist :) okay!! :D
 
@r9m I also tried another method but it seemed a bit troublesome.
 
r9m
3:13 PM
@Chris'ssistheartist the second part? That is giving me nightmares!! :O robjohn's proof is by far the most elegant approach!
 
@r9m well, robjohn is an expert in these questions.
 
r9m
@Chris'ssistheartist :D :D
 
Huy
4:05 PM
@evinda: Did you decide how to vote tomorrow yet?
 
4:44 PM
@r9m That's nice! Are you still working on the second part, or are you moving on?
@Chris'ssistheartist: I remember answering that at one point.
 
@robjohn The integral I just deleted?
 
@Chris'ssistheartist yes
 
@robjohn It's a question I newly created.
 
@Chris'ssistheartist really? I am sure that I have answered that in the past.
 
@robjohn That would be amazing if it's posted somewhere. Well, it's possible but I created it independetly of any other work. Let me know if you find it somewhere or anything similar.
 
4:48 PM
hi @mixedmath
 
@BalarkaSen hiya
 
what've you been thinking about?
 
I'm currently teaching high schoolers about number theory
that's been taking a lot of my time for the last few weeks, and for the next several weeks as well
 
ah. must be fun.
 
00:00 - 17:0017:00 - 00:00

« first day (1795 days earlier)      last day (3229 days later) »